OB/PEDS HESI COMBO

Ace your homework & exams now with Quizwiz!

The nurse is calculating the estimated date of confinement (EDC) using Ngele's rule for a client whose last menstrual period started on December 1. Which date is most accurate? A. August 1 B. August 10 C. September 3 D. September 8

D. September 8

Put the following actions in order to prevent hypotension in the pregnant client: 1. reposition the client 2. provide oxygen via face mask 3. increase IV fluid 4. call the healthcare provider

1. reposition the client 2. increase the IV fluid 3. provide oxygen via face mask 4. call the healthcare provider.

Which behavior would the nurse expect a two-year-old child to exhibit? A. build a house with blocks B. ride a tricycle C. display possessiveness of toys D. look at a picture book for 15 minutes

C. display possessiveness of toys Two-year old children are egocentric and unable to share with other children. (A, B, and D) are behaviors of a preschooler.

A 40 week gestation primigravada client is being induced with an oxytocin (pitocin) secondary infusion and complains pain in her lower back. Which intervention should the nurse implement? A. discontinue the oxytocin (Pitocin) infusion B. place the client in a semi-Fowler's position C. inform the healthcare provider D. apply firm pressure on the sacral area.

D. apply firm pressure on the sacral area

A primigravida client who is 5 cm dilated, 90% effaced, and at 0 station is requesting an epidural for pain relief. Which assessment finding is most important for the nurse to report to the healthcare provider? A. cervical dilation of 5 cm with 90% effacement B. WBC of 12,000/mm3 C. hemoglobin of 12 mg/dl and hematocrit of 38%. D. a platelet count of 67,000/mm3

D. a platelet count of 67,000/mm3. This low amount places the client at risk for bleeding from an epidural

The nurse is planning preconception care for a new female client. Which information should the nurse provide to the client? A. discuss various contraceptive methods to use until pregnancy is desired B. provide written or verbal information about prenatal care C. ask the client about risk factors associated with complications of pregnancy D. encourage healthy lifestyles for families desiring pregancy

D. encourage healthy lifestyles for families desiring pregancy

A woman who thinks she could be pregnant calls her neighbor, a nurse, to ask when she could use a home pregnancy test to diagnose pregnancy. Which response is appropriate? A. "A home pregnancy test can be used right after your first missed period." B. "These tests are most accurate after you have missed your second period." C. "Home pregnancy tests often give false positives and should not be trusted." D. "The test can provide accurate information when used right after ovulation."

A. "A home pregnancy test can be used right after your first missed period."

A pregnant client with mitral stenosis Class III is prescribed complete bedrest. The client asks the nurse, "Why must I stay in bed all the time?" Which response is best for the nurse to provide the client? A. "Complete bedrest decreases oxygen needs and demands on the heart muscle tissue." B. "We want your baby to be healthy, and this is the only way we can make sure that will happen." C. "I know you're upset. Would you like to talk about some things you could do while in bed." D. "Labor is difficult and you need to use this time to rest before you have to assume all child-caring duties."

A. "Complete bedrest decreases oxygen needs and demands on the heart muscle tissue."

A client at 32 weeks gestation is diagnosed with preeclampsia. Which assessment finding is most indicative of an impending convulsion? A. 3+ deep tendon reflexes and hyperclonus B. periorbital edema, flashing lights, and aura C. epigastric pain in the third trimester D. recent decreased urinary output

A. 3+ deep tendon reflexes and hyperclonus

As part of the physical assessment of children, the nurse observes and palpates the fontanels. Which child's fontanel finding should be reported to the healthcare provider? A. A 5-month old with failure to thrive that has a closed anterior fontanel B. a 24-month old with gastroenteritis that has a closed posterior fontanel. C. A 2-month old with chickenpox that has an open posterior fontanel. D. A 28-month old with hydrocephalus that has an open anterior fontanel.

A. A 5-month old with failure to thrive that has a closed anterior fontanel At six months of age the anterior fontanel should be open, and it should not be closed until approximately 18 months of age. (B and C) are normal findings. A child with hydrocphalus may have a delayed closing of the fontanel.

On admission to the prenatal clinic, a 23-year old woman tells the nurse that her last menstrual period began on February 15, and that previously her periods were regular. Her pregnancy test is positive. This client's expected date of delivery (EDD) would be A. November 22 B. November 8 C. December 22 D. October 22

A. November 22

A 3-month old infant develops oral thrush. Which pharmacologic agent should the nurse plan to administer for treatment of this disorder? A. Nystantin (Mycostatin) B. Nitroflurantoin (Macrodantin) C. Norfloxacin (Noroxin) D. Neomycin sulfate (Mycifradin)

A. Nystantin (Mycostatin) Nystantin is an antifungal drug that is effective in treating thrush, an oral fungal infection. (C and D) are not? indicated for the treatment of oral thrush.

Client teaching is an important part of the maternity nurse's role. Which factor has the greatest influence on successful teaching of the gravid client? A. The client's readiness to learn B. The client's educational background C. The order in which the information is presented D. The extent to which the pregnancy is planned

A. The client's readiness to learn

To take the vital signs of a 4-month old child, which order will give the most accurate results? A. Respiratory rate, heart rate, then rectal temperature B. Heart rate, rectal temperature, then respiratory rate. C. Rectal temperature, heart rate, then respiratory rate D. Rectal temperature, respiratory rate, then heart rate

A. Respiratory rate, heart rate, then rectal temperature The respiratory rate should be taken first in infants, since touching them or performing unpleasant procedures usually makes them cry, elevating the heart rate and making respirations difficult to count. Rectal temperature is the most invasive procedure, and is mot likely to precipitate crying, so should be done last.

A female teenager is taking oral tetracycline HCL (Achromycin V) for acne vulgaris. What is the most important instruction for the nurse to include in this client's teaching plan? A. Use sunscreen when lying by the pool. B. cleanse the skin at least 4 times a day. C. take the medication with a glass of milk D. menstrual periods may become irregular

A. Use sunscreen when lying by the pool. Photosensitivity is a common side effect of tetracycline HCL (Achromycin V) therapy. Severe sunburn can occur with minimal sun exposure and clients should be instructed to avoid sunlight and to use sunscreen. (B and D) are not related to tetracyline HCL (Achromycin V) therapy. (C) should be avoided because dairy products interfere with the absorption of tetracyclines.

Twenty four hours after admission to the newborn nursery, a full-term male infant develops localized edema on the right side of his head. The nurse knows that, in the newborn, an accumulation of blood between the periosteum and skull which does not cross the suture line in a newborn variation known as A. a cephalhematoma, caused by forceps trauma and may last up to 8 weeks B. a subarachnoid hematoma, which requires immediate drainage to prevent further complications C. molding, caused by pressure during labor and will disappear within 2 to 3 days D. a subdural hematoma which can result in lifelong damage

A. a cephalhematoma, caused by forceps trauma and may last up to 8 weeks

The nurse is performing a gestational age assessment on a full-term newborn during the first hour of transition using the Ballard (Dubowitz) scale. Based on this assessment, the nurse determines that the neonate has a maturity rating of 40 weeks. What findings should the nurse identify to determine if the neonate is small for gestational age (SGA)? Select all that apply A. admission weight of 4 pounds, 15 ounces (2244 grams) B. head to heel length of 17 inches (42.5 cm) C. Frontal occipital circumference of 12.5 in (31.25 cm) D. Skin smooth with visible veins and abundant vernix E. Anterior plantar crease and smooth heel surfaces F. Full flexion of all extremities in resting supine position

A. admission weight of 4 pounds, 15 ounces (2244 grams) B. head to heel length of 17 inches (42.5 cm) C. Frontal occipital circumference of 12.5 in (31.25 cm). The normal full-term appropriate for gestational age (AGA) newborn should fall between the measurement ranges of weight 6-9 pounds, length 19-21 inches, FOC 13-14 inches. This neonate's parameters plot below the 10% percentile, which indicate that the infant is SGA.

A 6-month old infant with congestive heart failure (CHF) is receiving digoxin elixer. Which observation by the nurse warrants immediate intervention? A. apical heart rate of 60. B. sweating across the forehead C. doesn't suck well D. respiratory rate of 30 breaths/minute

A. apical heart rate of 60. A heart rate of 60 is much lower than normal for a 6-month old and warrants immediate intervention. the normal heart rate for a 6-month old is 80-150 bpm when awake, and a rate of 70 while sleeping is considered within normal limits. (B and C) are expected symptoms of heart failure in an infant. (D) normal limits for an infant.

A woman who gave birth 48 hours ago is bottle-feeding her infant. During assessment, the nurse determines that both breasts are swollen, warm, and tender upon palpation. What action should the nurse take? A. apply cold compresses to both breasts for comfort. B. instruct the client to run warm water on her breasts C. wear a loose-fitting bra to prevent nipple irritation D. express small amounts of milk to relieve pressure

A. apply cold compresses to both breasts for comfort.

A vaginally delivered infant of an HIV positive mother is admitted to the newborn nursery. What intervention should the nurse perform first? A. bathe the infant with an antimicrobial soap B. measure the head and chest circumference C. obtain the infant's footprints D. administer vitamin K (AquaMEPHYTON)

A. bathe the infant with an antimicrobial soap. To reduce direct contact with the Human immuno-virus in blood and body fluids on the newborn's skin, a bath with an antimicrobial soap should be administered first.

The nurse is teaching a woman how to use her basal body temperature (BBT) pattern as a tool to assist her in conceiving a child. Which temperature pattern indicates the occurrence of ovulation, and therefore, the best time for intercourse to ensure conception? A. between the time the temperature falls and rises B. between 36 and 48 hours after the temperature rises C. when the temperature falls and remains low for 36 hours D. within 72 hours before the temperature falls

A. between the time the temperature falls and rises

A multigravida client at 41 weeks gestation presents in the labor and delivery unit after a non-stress test indicated that the fetus is experiencing some difficulties in utero. Which diagnostic test should the nurse prepare the client for additional information about fetal status? A. biophysical profile (BPP) B. ultrasound for fetal anomalies C. maternal serum alpha-fetoprotein (AF) screening D. percutaneous umbilical blood sampling (PUBS)

A. biophysical profile (BPP). This test provides data regarding fetal risk surveillance by examining 5 areas: fetal breathing movements, fetal movements, amniotic fluid volume, and fetal tone and heart rate. The client's gestation has progressed past the estimated date of confinement, so the major concern is fetal well-being related to the aging placenta.

A 4-year old boy was admitted to the emergency room with a fractured right ulna and a short arm cast was applied. When preparing the parents to take the child home, which discharge instruction has the highest priority? A. call the healthcare provider immediately if his nail beds appear blue. B. check his fingers hourly for the first 48 hours to see that he is able to move them without pain. C. Be sure your child's arm remains above his heart for the first 24 hours. D. Take his temperature every four hours for the next two days and call if an elevation is noted.

A. call the healthcare provider immediately if his nail beds appear blue. Cyanosis indicates impaired circulation to fingers and should be reported immediately. Although the actions described in (B, C, and D) may be indicated, they are implemented rather excessively - and might tend to frighten the parents. It is not necessary to check the child's ability to move his fingers hourly for two days. Elevating the arm above the heart helps to decrease swelling but (C) is stated in a frightening way. It is not necessary to take the child's temperature q4h unless indicated by other symptoms.

The nurse is providing discharge for a client who is 24 hours postpartum. The nurse explains to the client that her vaginal discharge will change from red to pink to white. The client asks, "What if I start having red bleeding after it changes?" What should the nurse instruct the client to do? A. reduce activity level and notify the healthcare provider B. go to bed and assume a knee-chest position C. massage the uterus and go to the emergency room d. do not worry as this is a normal occurance

A. reduce activity level and notify the healthcare provider

Which measurements should be used to accurately calculate a pediatric medication dosage? (Select all that apply) A. child's height and weight B. adult dosage of medication C. body surface area of child D. average adult's body surface area E. average pediatric dosage of medication F. nomogram determined mathematical constant.

A. child's height and weight C. body surface area of child F. nomogram determined mathematical constant. The most accurate calculations of pediatric dosages use the child's height and weight. The child's BSA is calculated using the square root of weight in kg times height in cm divided by 3600 or the square root of weight in lb times height in inches divided by 3131, then the child's BSA is multiplied by the recommended published dose per BSA. the nomogram is used to plot the child's height and weight, and the point at which they intersect is the BSA mathematical constant used to calculate the child's dose.

A four-year old girl continues to interrupt her mother during a routine clinic visit. the mother appears irritated with the child and asks the nurse, "Is this normal behavior for a child this age?" The nurse's response should be based on which information? A. children need to retain a sense of initiative without impinging on the rights and privileges of others. B. negative feelings of doubt and shame are characteristic of 4 year-old children C. Role conflict is a common problem of children this age. She is just wondering where she fits into society. D. At this age children compete and like to produce and carry through with tasks. She is just competing with her mother.

A. children need to retain a sense of initiative without impinging on the rights and privileges of others. Children ages 3-6 are in Erikson's "Initiative vs. Guilt" stage, which is characterized by vigorous, intrusive behavior, enterprise, and strong imagination. At this age, children develop a conscience and must learn to retain a sense of initiative without impinging on the rights of others. (B) describes the "Autonomy vs. Shame and Doubt" stage (1-3 years). (C) describes an adolescent (12-18) "Identify vs. Role Confusion" stage. (D) describes a child 6-12 years of age in the "Industry vs. Inferiority" stage.

A full term infant is admitted to the newborn nursery. After careful assessment, the nurse suspects that the infant may have an esophageal atresia. Which symptoms are this newborn likely to exhibit? A. choking, coughing, and cyanosis B. projectile vomiting and cyanosis C. apneic spells and grunting D. scaphoid abdomen and anorexia

A. choking, coughing and cyanosis

A full-term infant is admitted to the newborn nursery. After careful assessment, the nurse suspects that the infant may have an esophageal atresia. Which symptoms is this newborn likely to have exhibited? A. choking, coughing, and cyanosis B. projectile vomiting and cyanosis C. apneic spells and grunting D. scaphoid abdomen and anorexia

A. choking, coughing, and cyanosis (A) includes the "3 C's" of esophageal atresia caused by the overflow of secretions into the trachea. (B) is characteristic of pyloric stenosis in the infant. (C) could be due to prematurity or sepsis, and grunting is a sign of respiratory distress. (D) is characteristic of a diaphragmatic hernia.

A client at 28 weeks gestation calls the antepartal clinical and states that she is experiencing a small amount of vaginal bleeding which she describes as bright red. She further states that she is not experiencing any uterine contractions or abdominal pain. What instruction should the nurse provide? A. come to the clinic today for an ultrasound B. go immediately to the emergency room C. lie on your left side for about one hour and see if the bleeding stops D. bring a urine specimen to the lab tomorrow to determine if you have a urinary tract infection

A. come to the clinic today for an ultrasound. Third trimester painless bleeding is characteristic of a placenta previa. Bright red bleeding may be intermittent, occur in gushes, or be continuous. Rarely is the first incidence life-threatening, nor cause for hypovolemic shock. Diagnosis is confirmed by transabdominal ultrasound.

A multigravida client arrives at the labor and delivery unit and tells the nurse that her "bag of water" has broken. The nurse identifies the presence of meconium fluid on the perineum and determines the fetal heart rate is between 140-150 bpm. What action should the nurse implement next? A. complete a sterile vaginal exam B. take maternal temperature every 2 hours C. prepare for an immediate cesarean birth D. obtain sterile suction equipment

A. complete a sterile vaginal exam. This is done to determine the presence of a prolapsed umbilical cord.

A 2-year old child with Down syndrome is brought to the clinic for his regular physical examination. The nurse knows which problem is frequently associated with Down syndrome? A. congenital heart disease B. fragile x-chromosome C. trisomy 13 D. pyloric stenosis

A. congenital heart disease Congenital heart disease is the most common associated defect in children with Down syndrome. Trisomy 13 my have seemed possible since Down syndrome is a trisomal chromosomal abnormality o chromosome 21. Fragile x-chromosome is a sex-linked abnormality also causing mental retardation. Pyloric stenosis is not associated with Down syndrome.

A healthcare provider informs the charge nurse of a labor and delivery unit that a client is coming to the unit with suspected abruptio placentae. What findings should the charge nurse expect the client to demonstrate? (Select all that apply) A. dark, red vaginal bleeding B. lower back pain C. premature rupture of the membranes D. increased uterine irritability E. bilateral pitting edema F. a rigid abdomen

A. dark, red vaginal bleeding D. increased uterine irritability

A 35-year old primigravida client with severe preeclampsia is receiving magnesium sulfate via continuous IV infusion. Which assessment data would indicate to the nurse that the client is experiencing magnesium sulfate toxicity? A. deep tendon reflexes 2+ B. blood pressure 140/90 C. respiratory rate 18/min D. urine output 90 mL/4 hours

A. deep tendon reflexes 2+

A woman with type 2 diabetes mellitus becomes pregnant, and her oral hypoglycemic agents are discontinued. Which intervention is most important for the nurse to implement? A. Describe diet changes that can improve the management of her diabetes B. inform the client that oral hypoglycemic agents are teratogenic during pregnancy C. Demonstrate self-administration of insulin D. evaluate the client's ability to do glucose monitoring

A. describe diet changes that can improve the management of her diabetes

The parents of a 3-week old infant report that the child eats well but vomits after each feeding. what information is most important for the nurse to obtain? A. description of vomiting episodes in the past 24 hours B. number of wet diapers in last 24 hours C. feeding and sleep schedule D. amount of formula consumed during the past 24 hours

A. description of vomiting episodes in the past 24 hours A description of the vomiting episodes will assist the nurse in determining the reason for the symptoms, which may be helpful in developing a plan of care for this infant. (B and C) provide related information but are not as helpful as (A). (D) may be related to vomiting but the nurse should first obtain a better description of the vomiting episodes.

The nurse is caring for a woman with a previously diagnosed heart disease who is in the second stage of labor. Which assessment findings are of the greatest concern? A. edema, basilar rates, and an irregular pulse B. increased urinary output and tachycardia C. shortness of breath, bradycardia, and hypertension D. regular heart rate and hypertension

A. edema, basilar rates, and an irregular pulse

The nurse is having difficulty communicating with a hospitalized 6 year old child. Which approach by the nurse is the most helpful in establishing communication? A. engage the child through drawing pictures B. suggest that the parent read a book to the child C. provide paper and pencil for the child to keep a diary D. ask the parent if the child is always uncommunicable

A. engage the child through drawing pictures Drawing pictures is a valuable form of non-verbal communication. As the nurse and child look at the drawings, a verbal story can be told that projects the child's thinking. (B) may distract the child, but does not establish communication with the nurse. (C) is useful for an older child who is able to read and write. (D) is important, but engaging the child is more effective in establishing communication patterns.

A 6 year old is admitted to the pediatric unit after falling off a bicycle. Which intervention should the nurse implement to assist the child's adjustment to hospitalization? A. explain hospital schedules to the child, such as mealtimes. B. use terms, such as "honey" and "dear" to show a caring attitude. C. provide a list of rules that limits visitation of siblings in the hospital D. orient the parents to the hospital unit and refreshment areas.

A. explain hospital schedules to the child, such as mealtimes. Altered daily schedules and loss of rituals are upsetting to children and increase separation anxiety, and active sensitivity to the needs of children can minimize the negative effects of hospitalization. Explaining the hospital schedules and establishing an individual schedule familiarizes the child to the hospital environment and decreases anxiety. (B) depersonalizes the child who should be addressed by name. Family and sibling visitation should be recommended and encouraged without limitation. Although (D) should be implemented, the direct involvement of the school-aged child incorporates the child's sense of initiate and cooperation.

The healthcare provider prescribes terbutalne (Brethine) for a client in preterm labor. Before initiating this prescription, it is most important for the nurse to assess the client for which condition? A. gestational diabetes B. elevated blood pressure C. urinary tract infection D. swelling in lower extremities

A. gestational diabetes

The nurse attempts to help an unmarried teenager deal with her feelings following a spontaneous abortion at 8 weeks gestation. What type of emotional response should the nurse anticipate? A. grief related to her perceptions about the loss of this child B. Relief of ambivalent feelings experienced with this pregnancy C. Shock because she may not have realized that she was pregnant D. guilt because she had not followed her healthcare provider's instructions

A. grief related to her perceptions about the loss of this child

Which action should the nurse implement when preparing to measure the fundal height of a pregnant client? A. have the client empty her bladder B. request the client lie on her left side C. Perform Leopold's maneuvers first D. Give the client some cold juice to drink

A. have the client empty her bladder

A burned child is brought to the emergency room. In estimating the percentage of the body burned, the nurse uses a modified "Rule of Nines." Which part of a child's body is calculated as a larger percentage of total body surface than an adult? A. head and neck B. arms and chest C. legs and abdomen D. back and abdomen

A. head and neck A child's head and neck are proportionately larger to their body than an adults. The standard "Rule of Nine's" is inaccurate for determining burned body surface areas with children and must be modified for use with children. Specially designed charts for children are commonly used to determine body surface area involvement (B, C and D) are not proportionately different.

Immediately after birth, a newborn infant is suctioned, dried, and placed under a radiant warmer. The infant has spontaneous respirations and the nurse assesses an apical heart rate of 80 bpm and respirations of 20 breaths/min. What action should the nurse perform next? A. initiate positive pressure ventilation B. intervene after the one minute Apgar is assessed C. initiate CPR on the infant D. assess the infant's blood glucose level

A. initiate positive pressure ventilation because the infant's vital signs are not within the normal range and oxygen deprivation leads to cardiac depression in infants. The normal newborn pulse is 100-160 bpm and respirations are 40-60 breaths/minute.

When assessing a child with asthma, the nurse should expect intercostal retractions during A. inspiration B. coughing C. apneic episodes D. expiration

A. inspiration Intercostal retractions result from respiratory effort to draw air into restricted airways.

Which nurse intervention would be most helpful in relieving postpartum uterine contractions or "afterpains?" A. lying prone with a pillow on the abdomen B. using a breast pump C. massaging the abdomen D. giving oxytocic medications

A. lying prone with a pillow on the abdomen

Which finding in a 19 year old female client should trigger further assessment by the nurse? A. menstruation has not occurred B. reports no tetanus immunization since childhood C. denies having any wisdom teeth D. history of painful, inward growth on bottom of foot

A. menstruation has not occurred Menstruation is an expected secondary sex characteristic that occurs with pubescence and typically occurs by age 18 so (A) should prompt further investigation to determine the cause of this primary amenorrhea. Children receive tetanus as part of the DPT childhood immunization series, and a booster is not typically given until age 16. Wisdom teeth are the third molar teeth of the permanent dentition and are the last to erupt so (C) is a normal finding. (D) describes a plantar surface wart, harmless but painful because of the pressure with walking or standing.

A 3 week old newborn is brought to the clinic for follow after a home birth. the mother reports that her child bottle feeds for 5 minutes only and then falls asleep. the nurse auscultates a loud murmur characteristic of a ventricular septal defect (VSD) and finds the newborn is acyanotic with a respiratory rate of 64 breaths per minute. What instruction should the nurse provide the mother to ensure the infant is receiving adequate intake? (select all that apply) A. monitor the infant's weight and number of wet diapers per day. B. increase the infant's intake per feeding by 1-2 ounces per week C. Mix the dose of prophylactic antibiotic in a full bottle of formula D. allow the infant to rest and refeed on demand on every 2 hours E. use a softer nipple or increase the size of the nipple opening

A. monitor the infant's weight and number of wet diapers per day. B. increase the infant's intake per feeding by 1-2 ounces per week D. allow the infant to rest and refeed on demand on every 2 hours E. use a softer nipple or increase the size of the nipple opening Neonates who have VSD may fatigue quickly during feeding and ingest inadequate amounts. they should be monitored for weight gain and at least 6 wet diapers per day. A one-month old infant should ingest 2-4 ounces of formula per feeding and progress to about 30 ounces per day by 4 months of age. due to fatigue, the infant should rest, but feed at least every 2 hours to ensure adequate intake. A softer (preemie) nipple or a larger slit in the nipple helps to reduce the sucking effort and energy expenditure thus allowing the infant to ingest more with less effort. Antibiotic prophylaxis is recommended for infants with VSDs, but should not be mixed in a bottle of formula because it is difficult to ensure that the total dose is consumed.

When explaining "postpartum blues" to a client who is 1 day postpartum, which symptoms should the nurse include in the teaching plan? (Select all that apply) A. mood swings B. panic attacks C. tearfulness D. decreased need for sleep E. disinterest in the infant

A. mood swings C. tearfulness

Which menu selection by a child with celiac disease indicates to the nurse that the child understands necessary dietary considerations? A. oven-baked potato chips and cola. B. peanut butter and banana sandwich C. oatmeal-raisin cookies and milk D. graham crackers and fruit juice

A. oven-baked potato chips and cola. Celiac disease causes an intolerance to the protein gluten found in oats, rye, wheat and barley. the child should avoid any products containing these ingredients to avoid symptoms such as diarrhea. (A) is the selection which avoids all of these ingredients. (B, C and D) contain gluten in one form or another.

The nurse is assessing the neurovascular status of a child in Russell's traction. Which finding should the nurse report to the healthcare provider? A. pale bluish coloration of the toes B. skin is warm and dry to the touch C. toes are wiggled upon command D. capillary refill less than 3 seconds

A. pale bluish coloration of the toes Russell's skin traction is used for fractures of the femur in young children and adolescents, whose growth plates remain open and is applied to the lower leg using moleskin and elastic wrap bandages, which can compress the peroneal nerve and arteries that supply the foot. Assessment of adequate circulation, movement, and sensation of the toes and skin distal to the application is made to identify compromised blood flow so cyanosis should be reported immediately.

When discussing discipline with the mother of a 4 year old child, the nurse should include which guideline? A. parental control should be consistent B. children as young as 4 years rarely need reprimand or punishment C. Withdrawal of approval is effective D. Parents should enforce rigid rules to be followed without question.

A. parental control should be consistent Discipline should be a positive and necessary component of childrearing that is started in infancy and should teach socially acceptable behavior to help children protect themselves from danger, and channel undesirable behavior into constructive activity. Misbehavior may result from inconsistent rules or messages, so parental attention should be clear, reasonable, and consistent. (B and C) are not helpful to the child. Children need boundaries that are firm but not rigid.

The nurse receives a lab report stating a child with asthma has a theophylline level of 15 mcg/dl. What action will the nurse take? A. pass the information on in the report B. notify the healthcare provider because the value is high C. repeat the lab study because the value is too high. D. hold the next dose of theophylline

A. pass the information on in the report The therapeutic level of theophylline is 10-20 mcg/dL, so the child's level is within the therapeutic range. This information evaluates the prescribed therapy and should be communicated in the nurse's report. (B, C, and D) would be inappropriate actions in view of the laboratory finding.

A client in active labor is admitted with preeclampsia. Which assessment finding is most significant in planning this client's care? A. patellar reflex 4+ B. blood pressure 158/80 C. four-hour urine output 240 mL D. respiration 12/minute

A. patellar reflex 4+. a 4+ reflex in a client with pregnancy-induced hypertension indicates hyperreflexia, which is an indication of impending seizure.

The nurse is teaching a 12-year old male adolescent and his family about taking injections of growth hormone for idiopathic hypopituitarism. Which adverse symptoms, commonly associated with growth hormone therapy should the nurse plan to describe to the child and his family? A. polyuria and polydipsia B. lethargy and fatigue C. increased facial hair D. facial bone structure changes

A. polyuria and polydipsia Signs and symptoms of diabetes or hyperglycemia need to be reported. those receiving growth hormone should be monitored to detect elevated blood sugars and glucose intolerance. Lethargy and fatigue are associated with any number of health alterations, but is not associated with the growth hormone therapy. Increased facial hair and facial bone structure changes are normal changes that occur with 12-year old males.

A client receiving epidural anesthesia begins to experience nausea and becomes pale and clammy. What intervention should the nurse implement first? A. raise the foot of the bed B. assess for vaginal bleeding C. evaluate the fetal heart rate D. take the client's blood pressure

A. raise the foot of the bed. These symptoms are suggestive of hypotension which is a side effect of epidural anesthesia. Raising the foot of the bed (Trendelenburg position) will increase venous return and provide blood to the vital areas.

When evaluating the effectiveness of interventions to improve the nutritional status of an infant with gastro-esophageal reflux, which intervention is most important for the nurse to implement? A. record weight daily B. assess for signs of anemia C. document sleeping patterns D. teach parenting skills

A. record weight daily The most definitive measure of improved nutrition in an infant is obtaining the child's weight daily. (B, C, and D) may also be useful, but they are not as definitive as a daily weight measurement.

The nurse reviews the latest laboratory results for a child who received chemotherapy last week and identifies a reduced neutrophil count. Which nursing diagnosis has the highest priority for this child? A. risk for infection B. risk for hemorrhage C. altered skin integrity D. disturbance in body image

A. risk for infection Chemotherapy suppresses phagocytotic neutrophils and places the child at risk for infection which is the priority nursing diagnosis. (B, C and D) may be related to the care of a child receiving CT are not related to neutropenia.

A 4 week old premature infant has been receiving epoetin alfa (Epogen) for the last 3 weeks. Which assessment finding indicates to the nurse that the drug is effective? A. slowly increasing urinary output over the last week B. respiratory rate changes from the 40s to the 60s. C. changes in apical heart rate from the 180s to the 140s D. change in indirect bilirubin from 12 mg/dl to 8 mg/dl

A. slowly increasing urinary output over the last week

A 17 year old male student reports to the school clinic one morning for a scheduled health exam. He tells the nurse that he just finished football practice and is on his way to class. The nurse assesses his vital signs: temperature 100F, pulse 80, respirations 20, and BP 122/82. What is the best action for the nurse to take? A. tell the student to proceed directly to his regularly scheduled class. B. call the parent and suggest re-taking the student's temperature at home C. give the student a glass of cool fluids, then retake his temperature D. send the student to class, but re-verify his temperature after lunch

A. tell the student to proceed directly to his regularly scheduled class. This student has just completed football practice, and increased muscle activity increases body heat production. A temperature of 100F is normal for this student at this time. The student should attend class since no further nursing action is required. (B) would alarm the parents unnecessarily. (C) would provide a false reading of body temperature. (D) is unnecessary since these findings are within the normal limits.

A 42 week gestational client is receiving an intraenous infusion of oxytocin (pitocin) to augment early labor. The nurse should discontinue the oxytocin infusion for which pattern of contractions? A. transition labor with contractions every 2 minutes, lasting 90 seconds each. B. early labor with contractions every 5 minutes, lasting 40 seconds each C. Active labor with contractions every 31 minutes, lasting 60 seconds each D. Active labor with contractions every 2-3 minutes, lasting 70-80 seconds each

A. transition labor with contractions every 2 minutes, lasting 90 seconds each. When oxytocin causes uterine hyperstimulation as evidence by inadequate resting time between contractions, the oxytocin infusion should be discontinued because placental perfusion is impeded.

The nurse is counseling a couple who has sought information about conceiving. For teaching purposes, the nurse should know that ovulation usually occurs A. two weeks before menstruation B. immediately after menstruation C. immediately before menstruation D. three weeks before menstruation

A. two weeks before menstruation

The clinic nurse is taking the history for a new 6 month old client. The mother reports that she took a great deal of aspirin while pregnant. Which assessment should the nurse obtain? A. type of reaction to loud noises B. any surgeries on the ears since birth C. drainage from the infant's ears D. number of ear infections since birth

A. type of reaction to loud noises Ototoxicity diminishes hearing acuity and causes symptoms of tinnitus and vertigo in older children who can express subjective symptoms, so assessing an infant's reaction to loud noises helps to determine an infant's risk for a hearing deficit related to a history of the mother taking an ototoxic drug, such as aspirin, while pregnant. (B, C, and D) are not associated with exposure to aspirin in utero.

The nurse is giving a liquid iron preparation to a 3 year old child. Which technique should the nurse implement to engage the child's cooperation? A. use a colorful straw B. mix the medication in water C. administer the medication using an oral syringe D. ask the pharmacy to provide an enteric tablet.

A. use a colorful straw A liquid iron preparation administered through a straw may help the child to accept the medication since young children consider drinking from a colorful straw fun. (B) may cause staining of the child's teeth. (C) is often used if the child is uncooperative. (D) is ineffective and should be requested from the healthcare provider.

To assess the effectiveness of an analgesic administered to a 4 year old, what intervention is best for the nurse to implement? A. use a happy face/sad face pain scale B. ask the mother if she thinks the analgesic is working C. assess for changes in the child's vital signs D. teach the child to point to a numeric pain scale

A. use a happy face/sad face pain scale A 4 year old can readily identify with simple pictures to show the nurse how he/she is feeling. (B) could be used to validate what the child is telling the nurse via the "faces" pain scale, but it is best to elicit the child's assessment of his/her pain level. (C) may not accurately reflect the effectiveness of pain medication as they can also be affected by other variables, such as fear. (D) requires abstract number skills beyond the level of a 4 year old.

The nurse is assessing a 3 day old infant with a cephaloheatoma in the newborn nursery. Which assessment finding should the nurse report to the healthcare provider? A. Yellowish tinge to the skin B. Babinski reflex present bilaterally C. pink papular rash on the face D. Moro reflex noted after a loud noise

A. yellowish tinge to the skin. Cephalohematomas are characterized by bleeding between the bone and its covering, the periosteum. Due to the breakdown of the red blood cells within a hematoma, the infant is at a greater risk for jaundice so it should be reported.

Which maternal behavior is the nurse most likely to see when a new mother receives her infant for the first time? A. she eagerly reaches for the infant, undresses the infant, and examines the infant completely. B. Her arms and hands receive the infant and she then traces the infant's profile with her fingertips C. Her arms and hands receive the infant and she then cuddles the infant to her own body. D. She eagerly reaches for the infant and then holds the infant close to her own body

B. Her arms and hands receive the infant and she then traces the infant's profile with her fingertips

A female client with insulin-dependent diabetes arrives at the clinic seeking a plan to get pregnant in approximately 6 months. She tells the nurse that she wants to have an uncomplicated pregnancy and a healthy baby. What information should the nurse share with the client? A. "Your current dose of insulin should be maintained throughout your pregnancy." B. "Maintain blood sugar levels in a constant range within normal limits during pregnancy." C. "The course and outcome of your pregnancy is not an achievable goal with diabetes." D. "Expect an increase in insulin dosages by 5 units/wk during the first trimester."

B. "Maintain blood sugar levels in a constant range within normal limits during pregnancy."

A new mother asks the nurse, "How do I know that my daughter is getting enough breast milk?" Which explanation will the nurse provide? A. "weigh the baby daily, and if she is gaining weight, she is eating enough." B. "your milk is sufficient if the baby is voiding pale straw-colored urine 6-10 times/day." C. "Offer the baby extra bottle milk after her feeding and see if she is still hungry." D. "If you're concerned, you might consider bottle feeding so that you can monitor her intake."

B. "your milk is sufficient if the baby is voiding pale straw-colored urine 6-10 times/day."

The mother of a 6-month old asks the nurse when her baby will get the first measles, mumps, and rubella (MMR) vaccine. Based on the recommended childhood immunization schedule published by the Centers for Disease Control, which response is accurate? A. 3-6 months B. 12-15 months C. 18-24 months D. 4-6 years

B. 12-15 months The first measles, mumps, and rubella (MMR) vaccine should be given no sooner than 12 months of age, and ideally between 12 and 15 months of age. Children 3-6 months should not receive the vaccine due to the presence of maternal antibodies. MMR is not routinely administered at 18-24 months, but other immunizations, such as DTaP and Hepatitis B may be given at that time. The second dose of MMR is routinely administered at 4-6 years, provided that at least 4 weeks have elapsed since the first dose, and if both doses were administered beginning at or after 12 months.

A 6-month old boy and his mother are at the healthcare provider's office for a well-baby checkup and routine immunizations. the healthcare provider recommends to the mother that the child receive an influenza vaccine. What medications should the nurse plan to administer today? A. The routine immunizations and schedule another appointment to administer the influenza vaccine B. All the immunizations with the influenza vaccine given at a separate site from any other injection. C. the influenza vaccine and schedule another appointment to administer the immunizations D. The influenza vaccine and the polio vaccine and schedule another appointment to administer the remaining immunizations.

B. All the immunizations with the influenza vaccine given at a separate site from any other injection. At 6 months of age, the routine immunizations include Hepatitis B, DTaP, Hib (Haemophilus influenza type b), PCV (Pneumococcal), IPV (inactive poliovirus) and influenza. the influenza vaccine should be given at a separate site from any other injection. Scheduling a return visit increases the risk that the mother will not bring the child back for the immunizations.

The nurse caring for a laboring client encourages her to void at least q2h, and records each time the client empties her bladder. What is the primary reason for implementing this nursing intervention? A. emptying the bladder during delivery is difficult because of the position of the presenting fetal part. B. An over-distending bladder could be traumatized during labor, as well as prolong the progress of labor C. urine specimens for glucose and protein must be obtained at certain intervals throughout labor. D. frequent voiding minimizes the need for catheterization which increases the chance of bladder infection

B. An over-distending bladder could be traumatized during labor, as well as prolong the progress of labor

The nurse is assessing a 13-year old girl with suspected hyperthyroidism. Which question is most important for the nurse to ask her during the admission interview? A. Have you lost any weight in the last month? B. Are you experiencing any type of nervousness? C. When was the last time you took synthroid? D. Are you having any problems with your vision?

B. Are you experiencing any type of nervousness? Assessing the client's physiological state upon admission is a priority, and nervousness, apprehension, hyperexcitability, and palpitations are signs of hyperthyroidism. Weight loss (even with a hearty appetite) occurs in those with hyperthyroidism, but assessing the client's neurological state has a higher priority. Hormone replacement is not administered to a client who is already producing too much thyroid. The client may have exophthalmus (bulging eyes), but hyperthyroidism does not cause vision problems.

During routine screening at a school clinic, an otoscope examination of a child's ear reveals a tympanic membrane that is pearly gray, slightly bulging, and not movable. What action should the nurse take next? A. No action required, as this is an expected finding for a school-aged child B. Ask the child if he/she has had a cold, runny nose, or any ear pain lately. C. Send a note home advising the parents to have the child evaluated by a healthcare provider as soon as possible. d. Call the parents and have them take the child home from school for the rest of the day.

B. Ask the child if he/she has had a cold, runny nose, or any ear pain lately. More information is needed to interpret these findings. The tympanic membrane is normally pearly gray, not bulging, and moves when the client blows against resistance or a small puff of air is blown into the ear canal. Since this child's findings are not completely normal, further assessment of history and related signs and symptoms is indicated for accurate interpretation of the findings. (A), (C), and (D) are inappropriate actions based on the data obtained from the otoscope examination.

The nurse is counseling a woman who wants to become pregnant. The woman tells the nurse that she has a 36-day menstrual cycle and the first day of her last menstrual period was January 8. The nurse correctly calculates that the woman's next fertile period will be A. January 14-15 B. January 22-23 C. January 30-31 D. February 6-7

B. January 30-31. This woman can expect her next period to begin 36 days from the first day of her last menstrual period - the cycle begins at the first day of the cycle and continues to the first day of the next cycle. Her next period would, therefore, began on February 13. Ovulation occurs 14 days before the first day of the menstrual period. Therefore, ovulation for this woman would occur January 31.

A pregnant client tells the nurse that the first day of her last menstrual period was August 2, 2006. Based on Nagele's rule, what is the estimated date of delivery? A. April 25, 2007 B. May 9, 2007 C. May 29, 2007 D. June 2, 2007

B. May 9, 2007

All of the following interventions can be used to evaluate the effectiveness of nursing and medical interventions used to treat diarrhea. Which intervention is least useful in the nurse's evaluation of a 20-month-old child. A. weighing diapers B. assessing fontanels C. checking skin turgor D. observing mucous membranes for moisture

B. assessing fontanels All of these interventions evaluate fluid status in infants. But, now old is this child? Posterior fontanel closes at 2 months and anterior fontanels close by 18 months of age. Remember normal growth and development

A client with gestational hypertension is in active labor and receiving an infusion of magnesium sulfate. Which drug should the nurse have available for signs of potential toxicity? A. oxytocin (pitocin) B. calcium gluconate C. terbutaline (Brethine) D. naloxone (narcan)

B. calcium gluconate

Which growth and development characteristic should the nurse consider when monitoring the effects of a topical medication for an infant? A. a lower sensitivity reaction to skin irritants B. a thin stratum corneum that increases topical absorption C. a smaller percentage of muscle mass D. a greater body surface area that requires larger doses

B. a thin stratum corneum that increases topical absorption infants have a thin outer skin later (stratum corneum), so the nurse should monitor the infant for a prompt onset and response to the application of topical medication.

The nurse is assigning care for a 4-year old child with otitis media and is concerned about the child's increasing temperature over the past 24 hours. When planning care for this child, it is important for the nurse to consider that: A. only an RN should be assigned to monitor this child's temperature. B. a tympanic measurement of temperature will provide the most accurate reading C. the licensed practical nurse should be instructed to obtain rectal temperatures on this child D. the healthcare provider should be asked to prescribe the method for measurement of the child's temperatures.

B. a tympanic measurement of temperature will provide the most accurate reading A tympanic membrane sensor is an excellent site because both the eardrum and hypothalamus (temperature-regulating center) are perfused by the same circulation. The sensor is unaffected by cerumen and the presence of suppurative or unsuppurative otitis media does not affect measurement. Rule of thumb for management-sterile procedures should be assigned to licensed personnel. Management skills will be tested on the NCLEX. An RN is not required to do this. Rectal temperature management is less accurate because of the possibility of stool in the rectum. It is unnecessary to contacted the healthcare provider.

At 8am, the unlicensed assistive personnel (UAP) informs the charge nurse that a female adolescent client with acute glomerulonephritis has a blood pressure of 210/110. The 4am BP reading was 170/88. The client reports to the UAP that she is upset because her boyfriend did not visit last night. What action should the nurse take? A. give the client her 9am prescription for an oral diuretic early. B. administer PRN prescription of nifedipine (Procardia) sublingually. C. notify the healthcare provider and inform the nursing supervisor of the client's condition D. attempt to calm the client and retake the BP in 30 minutes.

B. administer PRN prescription of nifedipine (Procardia) sublingually. Sublingual Procardia lowers BP very quickly, and this should be done first. (A) may also be done, but oral diuretics do not work as rapidly as the sublingual antihypertensive. When notifying the healthcare provider, the first thing he/she will want to know is if the PRN antihypertensive has been administered. (D) does not consider the seriousness of this finding. The nurse should stay with the client until the BP is reduced.

The nurse is caring for a 12-year old with Syndrome of Inappropriate Antidiuretic Hormone (SIADH). This child should be carefully assessed for which complication? A. poor skin turgor resulting from dehydration B. changes in LOC C. premature aging as the disease progresses D. severe edema from an excess of water and sodium

B. changes in LOC The child must be monitored for signs and symptoms of hyponatremia, which creates secondary CNS alterations, such as changes in LOC, seizure and coma. Fluid overload occurs with SIADH not dehydration (which occurs with diabetes insipidus). Premature aging is caused by hypersecretion of growth hormone, not SIADH. Severe edema is not found in children with SIADH because edema is caused by an excess of both water and sodium.

A 3-year old boy is brought to the emergency room because he swallowed an entire bottle of children's vitamin pills. Which intervention should the nurse implement first? A. insert NG tube for gastric lavage B. determine the child's pulse and respirations C. assess the child's LOC D. administer an IV D5/0.25 NS as prescribed

B. determine the child's pulse and respirations The most important principle in dealing with a poisoning is to treat the child first, not the poison. Initiate immediate life support measures with assessment of vital signs, in particular, respirations. Inserting an airway or initiating mechanical ventilation may be necessary. Assessment and identification of the poison should occur prior to insertion of an NG tube. (C and D) should occur after assessing the airway.

the mother of a preschool-aged client asks the nurse if it is all right to administer Pepto Bismal to her son when he 'has a tummy ache." After reminding the mother to check the label of all OTC drugs for the presence of aspirin, which instruction should the nurse include when replying to this mother's question? A. if the child's tongue darkness, discontinue the Pepto Bismal immediately B. do not give if the child has chickenpox, the flu, or any other viral illness. C. avoid the use of Pepto Bismal until the child is at least 16 years old. D. Pepto Bismal may cause a rebound hyperactivity, worsening the "tummy-ache."

B. do not give if the child has chickenpox, the flu, or any other viral illness. Pepto Bismal contains aspirin and there is the potential of Reye's syndrome. A dark tongue is a common effect of Pepto Bismal and does not warrant discontinuation. Pepto Bismal can be used by children. Pepto Bismal does not cause rebound hyperactivity, which is a complication of antacids containing calcium.

A client who has an autosomal dominant inherited disorder is exploring family planning options and the risk of transmission of the disorder to the infant. The nurse's response should be based on what information? A. males inherit the disorder with a greater frequency than females B. each pregnancy carries a 50% chance of inheriting the disorder C. the disorder occurs in 25% of pregnancies D. all children will be carriers of the disorder

B. each pregnancy carries a 50% chance of inheriting the disorder

A client in active labor complains of cramps in her leg. What intervention should the nurse implement? A. ask if she takes a daily calcium tablet B. extend the leg and dorsiflex the foot C. lower the leg off the side of the bed D. elevate the leg above the heart

B. extend the leg and dorsiflex the foot. "Toes to the nose"

During discharge teaching of a child with juvenile rheumatoid arthritis, the nurse should stress to the parents the importance of obtaining which diagnostic testing? A. hearing test B. eye exam C. chest x-ray D. fasting blood glucose test

B. eye exam Visual changes leading to blindness can occur in children wit JRA. Regular eye exams can help to prevent this complication. (A, C, and D) are not routinely necessary for management of JRA.

A primigravida at 40 weeks gestation is receiving oxytocin (Pitocin) to augment labor. Which adverse effect should the nurse monitor for during the infusion of Pitocin? A. dehydration B. hyperstimulation C. galactorrhea D. fetal tachycardia

B. hyperstimulation. Pitocin causes the uterine myofibril to contract, so unless the infusion is closely monitored, the client is at risk for hyperstimulation which can lead to tetanic contractions, uterine rupture, and fetal distress or demise.

A full term infant is transferred to the nursery from labor and delivery. Which information is most important for the nurse to receive when planning immediate care for the newborn? A. length of labor and method of delivery B. infant's condition at birth and treatment received C. feeding method chosen by the parents D. history of drugs given to the mother during labor

B. infant's condition at birth and treatment received

A client who is attending antepartum classes asks the nurse why her healthcare provider has prescribed iron tablets. The nurse's response is based on what knowledge? A. supplementary iron is more efficiently utilized during pregnancy B. it is difficult to consume 18 mg of additional iron by diet alone. C. iron absorption is decreased in the GI tract during pregnancy D. iron is needed to prevent megaloblastic anemia in the last trimester

B. it is difficult to consume 18 mg of additional iron by diet alone.

During a prenatal visit, the nurse discusses with a client the effects of smoking on the fetus. When compared with nonsmokers, mothers who smoke during pregnancy tend to produce infants who have A. lower Apgar scores B. lower birth weights C. respiratory distress D. a higher rate of congenital anomalies

B. lower birth weights

A 30 year old gravida 2, para 1 client is admitted to the hospital at 26 weeks gestation in preterm labor. She is given a dose of terbutaline sulfate (Brethine) 0.25 mg SQ. Which assessment is the highest priority for the nurse to monitor during the adminstration of this drug? A. maternal blood pressure and respirations B. maternal and fetal heart rates C. hourly urinary output D. deep tendon reflexes

B. maternal and fetal heart rates

A 30 year old gravida, 2 para 1 client is admitted to the hospital at 26 weeks gestation in preterm labor. She is started on an IV solution of terbutaline (Brethine). Which assessment is the highest priority for the nurse to monitor during the administration of this drug? A. maternal blood pressure and respirations B. maternal and fetal heart rates C. hourly urinary output D. deep tendon reflexes

B. maternal and fetal heart rates

A 24-hour old newborn has a pink papular rash with vesicles superimposed on the thorax, back and abdomen. What action should the nurse implement? A. notify the healthcare provider B. move the newborn to an isolation nursery C. document the finding in the infant's record D. obtain a culture of the vesicles

C. document the finding in the infant's record. Erythema Toxicum (or erythema neonatorium) is a newborn rash that is commonly referred to as "flea bites." but is a normal finding that is documented in the infant's record and requires no further action.

The nurse identifies crepitus when examining the chest of the newborn who was delivered vaginally. Which further assessment should the nurse perform? A. elicit a positive scarf sign on the affected side B. observe for an asymmetrical Moro (startle) reflex C. Watch for swelling of fingers on the affected side D. Note paralysis of affected extremity and muscles

B. observe for an asymmetrical Moro (startle) reflex. The most common neonatal birth trauma due to a vaginal delivery is fracture of the clavicle. Although an infant may be asymptomatic, a fractured clavicle should be suspected if the infant has limited use of the affected arm, malposition of the arm, an asymmetric Moro reflex, crepitus over the clavicle, focal swelling or tenderness, or cries when the arm is moved.

The nurse is planning care for school aged children at a community care center. Which activity is best for the children? A. building model airplanes B. playing follow-the-leader C. stringing large and small beads D. Playing with playdough and clay

B. playing follow-the-leader School-aged children strive for independence and productivity (Erikson's Industry vs. Inferiority) and enjoy individual and group activities related to real life situations, such as playing follow the leader. (A) is an individual activity that could contribute to feelings of inferiority and inadequacy if the task if too complex, although school-aged children enjoy crafts. (C and D) are more appropriate for pre-school children.

A nurse provides the parents with information on health maintenance for their child with sickle cell disease. Which information reflected by the parents indicates understanding of the child's care? A. daily iron supplements should be given B. plenty of fluids should be consumed daily C. immunizations should be delayed for a few years D. protective equipment should be worn for contact sports.

B. plenty of fluids should be consumed daily Adequate fluid intake decreases the viscosity of the blood which affects the incidence of vasocclusive crisis. (A and D) are not commonly indicated for a child with sickle cell disease. A routine immunization schedule is recommended for a child with SCD because of their increased susceptibility to infection that predisposes to sickling phenomena.

A six-month-old returns from surgery with elbow restraints in place. What nursing care should be included when caring for any restrained child? A. keep restraints on at all times. B. remove restraints one at a time and provide range of motion exercises C. Remove all restraints simultaneously and provide lay activities D. renew the healthcare provider's prescription for restraints every 72 hours.

B. remove restraints one at a time and provide range of motion exercises Removing restraints one at a time is safer than removing all of them at once. The child needs to exercise and should not be kept in restraints at all times. The renewal of the healthcare provider's prescription varies with hospitals and it does not really answer the question.

The nurse should explain to a 30 year old gravid client that alpha fetoprotein testing is recommended for which purpose? A. detect cardiovascular disorders B. screen for neural tube defects C. monitor for placental functioning D. assess for maternal pre-eclampsia

B. screen for neural tube defects

A 3 year old client with sickle cell anemia is admitted to the Emergency Department with abdominal pain. The nurse palpates an enlarged liver, an x-ray reveals an enlarged spleen, and a CBC reveals anemia. these findings indicate which type of crisis? A. Aplastic B. sequestration C. hyperhemolytic D. vaso-occlusive

B. sequestration The findings support a sequestration crisis, where blood pools in the spleen, and is characterized by abdominal pain and anemia. (A and C) crises produce anemia but no abdominal pain or splenic enlargement. (D) crisis may produce abdominal pain but no splenic enlargement or exacerbation of anemia.

A 5-month-old is admitted to the hospital with vomiting and diarrhea. The pediatrician prescribed dextrose 5% and 0.25% normal saline with 2 mEq KCI/100 mL to be infused at 25mL/hr. Prior to initiating the infusion, the nurse should obtain which assessment finding? A. frequency of emesis is the last 8 hours B. serum BUN and creatinine levels C. current blood sugar level D. appearance of the stool

B. serum BUN and creatinine levels Regardless of a client's age, adequate renal function must be present before adding potassium to IV fluids. (A) is important in determining the need for fluid replacement. (C) is not indicated. (D) is useful information, but will not impact administration of the prescribed IV solution.

The vital signs of a 4 year old child with polyuria are BP 80/40, pulse 118, and respirations 24. the child's pedal pulses are present with a volume of +1 and no edema is observed. What action should the nurse implement first? A. insert an indwelling urinary catheter B. start an IV infusion of normal saline C. send a specimen to the lab for urinalysis D. document the child's vital signs and pulses

B. start an IV infusion of normal saline the current vital sign readings and the decreased peripheral pulse volume indicate that the child is experiencing fluid volume deficit due to the polyuria, so the priority action is to restore fluid volume. (A) is useful in obtaining a precise urine output measure, but is a lower priority than restoring fluid volume at this time. (C) is not indicated based on the current assessment data, and (D) does not recognize the need for immediate action to combat the fluid volume deficit

The nurse is preparing a health teaching program for parents of toddlers and preschoolers and plans to include information about prevention of accidental poisonings. It is most important for the nurse to include which instructions? A. tell children they should not taste anything but food B. store all toxic agents and medicines in locked cabinets. C. Provide special play areas in the house and restrict play in other areas. D. punish children i they open cabinets that contain household chemicals.

B. store all toxic agents and medicines in locked cabinets.

Which class of antiinfective drugs is contraindicated for use in children under 8 years of age? A. aminoglycosides B. tetracyclines C. penicillins D. quinolones

B. tetracyclines Tetracyclines cause enamel hypoplasia and tooth discoloration in children under 8 years of age. (A, C and D) are not contraindicated for use in children.

A full-term infant is transferred to the nursery from labor and delivery. Which information is most important for the nurse to receive when planning immediate care for the newborn? A. the length of labor and method of delivery B. the infant's condition at birth and treatment received C. the feeding method chosen by the parents D. the history of drugs given to the mother during labor

B. the infant's condition at birth and treatment received

A premature newborn girl, born 24 hours ago, is diagnosed with a patent ductus arteriosus (PDA) and placed under an oxygen hood at 35%. The parents visit the nursery and ask to hold her. Which response should the nurse provide to the parents? A. Studies have shown that handling a sick newborn is not good for the baby and upsets the parents. B. the oxygen hood is holding the baby's oxygen level just at the point which is needed. You may stroke and talk to her. C. Since your baby has been doing well under oxygen for 24 hours, I can let you hold the baby without oxygen. D. You can hold the baby with the oxygen blowing on the baby's face since the level is very close to room air.

B. the oxygen hood is holding the baby's oxygen level just at the point which is needed. You may stroke and talk to her. The baby is at 35% which is much more than room air (21%) and at this time the baby should not be moved from under the hood. the nurse should offer the parents an alternative such as to stroke and reassure the infant. Holding sick babies benefits the infant and the parents, but the first consideration now has to be the infant's oxygenation. The nurse should not take the baby out from under the hood without a prescription from the healthcare provider, as this could severely compromise the infant. A PO2 of 35% cannot be readily achieved with "blow by" oxygen.

The nurse assigning care for a 5-year old with otitis media is concerned about the child's increasing temperature over the past 24 hours. Which statement is accurate and should be considered when planning care for the remainder of the shift? A. An RN should be assigned to take temperatures frequently. B. tympanic and oral temperatures are equally accurate C. the PN should take rectal temperatures on this child. D. The pediatrician should decide how to assess the temperature

B. tympanic and oral temperatures are equally accurate A tympanic membrane sensor approximates core temperatures because the hypothalamus and eardrum are perfused by the same circulation. Tympanic readings obtained using proper technique correlated moderately to strongly with oral temperatures in recent research studies. The sensor is unaffected by the cerumen or the presence of suppurative or unsuppurative otitis media. A RN is not required to take the child's temperature, but must assess readings received from assistive personnel. Although rectal readings are highly accurate, such an invasive procedure is unnecessary and it is not necessary to contact the pediatrician.

Which action by the nurse is most helpful in communicating with a preschool-aged child? A. speak clearly and directly to the child B. use a doll to play and communicate C. approach when a parent is not present D. play a board game with the child.

B. use a doll to play and communicate Communicating through play with a doll or other toy gives time for the child to feel comfortable with a stranger. (A) may frighten some children and is usually not as effective as (B). to provide security and comfort, preschool aged children should be approached when a parent is present. (D) is too advanced for a preschooler.

The nurse is teaching a mother to give 4 mL of liquid antibiotic to a 10 month old infant. Which statement by the parent indicates a need for further teaching? A. I will give this antibiotic to my child until it is finished B. using a teaspoon will help me measure this correctly. C. I will call the clinic if my child develops a rash or itching D. my baby should begin to eel better within a few days

B. using a teaspoon will help me measure this correctly. The prescribed medication is 4 mL per dosage and is measured with the most accuracy using a syringe, so if the parent uses a teaspoon, which is equivalent to 5 mL, further teaching is indicated. (A, C and D) indicate correct understanding and require no further intervention by the nurse.

A new mother who has just had her first baby says to the nurse, "I saw the baby in the recovery room. She sure has a funny looking head" Which response by the nurse is best? A. "This is not an unusual shaped head, especially for a first baby." B. "It may look funny to you, but newborn babies are often born with heads like your baby's." C. "That is normal, the head will return to a round shape within 7 to 10 days." D. "Your pelvis was too small, so the baby's head had to adjust to the birth canal."

C. "That is normal, the head will return to a round shape within 7 to 10 days."

A three-month old boy weighing 10 lbs, 15 oz has an axillary temperature of 98.9 degrees F. The nurse determines the daily caloric need for this child is approximately: A. 400 cal/day B. 500 cal/day C. 600 cal/day D. 700 cal/day

C. 600 cal/day 10 lbs, 15 oz = 10.9 lbs. Convert lbs to kg by dividing pounds by 2.2 (10.9/2.2) = 4.954 kg rounded to 5 kg. an infant requires 108 cal/kg/day (108 x 5 = 540 cal/day). However this infant requires 10% more calories because he has one degree temperature elevation. 10% of 540 is 54 and 54 + 540 = 594. This infant will require approximately 600 calories/day. Remember that a temperature elevation necessitates consumption of more calories.

Which restraint should be used for a toddler after a cleft palate repair? A. clove hitch B. Mummy C. elbow D. jacket

C. elbow Elbow restraints Elbow restraints prevent children from bending their arms and bringing their hands to the oral surgical site. A clove hitch restrains the hands, but the child can bend and bring their head to their hands. A mummy restraint is used during procedures. A jacket restraint restrains the body torso and is not appropriate.

The nurse is teaching the parents of a 5 year old with cystic fibrosis about respiratory treatments. Which statement indicates to the nurse that the parents understand? A. perform postural drainage before starting aerosol therapy B. Give respiratory treatments when the child is coughing a lot C. Administer aerosol therapy followed by postural drainage before meals D. ensure respiratory therapy is done daily during any respiratory infection

C. Administer aerosol therapy followed by postural drainage before meals Postural drainage for a child with cystic fibrosis is most effective when performed after nebulization and before meals, or at least 1 hour after eating to prevent N/V. Postural drainage uses gravity to promote mucous removal after nebulization treatments which open the airways. Pulmonary toileting or respiratory treatments should be given 3 to 4 times daily, not episodically.

The total bilirubin level of a 36 hour, breastfeeding newborn is 14 mg/dl. Based on this finding, which intervention should the nurse implement? A. provide phototherapy for 30 minutes q8h B. feed the newborn sterile water hourly C. encourage the mother to breastfeed frequently D. assess the newborn's blood glucose level

C. encourage the mother to breastfeed frequently

A client at 32 weeks gestation is diagnosed with preeclampsia. Which assessment is most indicative of an impending convulsion? A. 3+ deep tendon reflexes B. periorbital edema C. epigastric pain D. decreased urine output

C. epigastric pain. Epigastric pain is indicative of an edematous liver or pancreas which is an early warning sign of an impending convulsion (eclampsia) and requires immediate attention.

The nurse is teaching care of the newborn to a group of prospective parents and describes the need for administering antibiotic ointment into the eyes of the newborn. Which infectious organism will this treatment prevent from harming the infant? A. Herpes B. Staphylococcus C. Gonorrhea D. Syphilis

C. Gonorrhea. Erythromycin ointment is instilled into the lower conjunctiva of each eye within 2 hours after birth to prevent ophthalmica neonatorum, and infection caused by gonorrhea, and including conjunctivitis,, an infection caused by chlamydia. The infant may be exposed to these bacteria when passing through the birth canal.

The nurse instructs a laboring client to use accelerated-blow breathing. The client begins to complain of tingling fingers and dizziness. What action should the nurse take? A. administer oxygen by face mask B. notify the healthcare provider of the client's symptoms C. have the client breathe into her cupped hands D. check the client's blood pressure and fetal heart rate

C. Have the client breathe into her cupped hands.

A 38-week primagravida who works as a secretary and sits at a computer 8 hours each day tells the nurse that her feet have begun to swell. Which instruction would be most effective in preventing pooling of blood in the lower extremities? A. Wear support stockings B. Reduce salt in her diet C. Move about every hour D. Avoid constrictive clothing

C. Move about every hour

A client is admitted with the diagnosis of total placenta previa. Which finding is most important for the nurse to report to the healthcare provider immediately? A. heart rate of 100 bpm B. variable fetal heart rate C. onset of uterine contractions D. burning on urination

C. onset of uterine contractions. Total (complete) placenta previa involves the placenta covering the entire cervical os (opening). The onset of uterine contractions places the client at risk for dilation and placental separation, which causes painless hemorrhaging.

A preschool-age child who is hospitalized for hypospadias repair is most strongly influenced by which behavior? A. ability to communicate verbally B. response to separation from family C. concern for body integrity D. socialization with other children

C. concern for body integrity The preschooler's major stressor is concern for his body integrity. He fears that his "insides will leak out". A child undergoing surgery to his genitalia is even more concerned about body integrity. the preschooler is quite verbal so comprehension of the words he uses or hears may be inaccurate, while his imagination and fears may fantasize the reality. (B) is a concern for all children, but of most concern to the toddler. (D) is not a prime concern in this situation.

When preparing a class on newborn care for expectant parents, what content should the nurse teach concerning the newborn infant born at term gestation? A. Milia are red marks made by forceps and will disappear within 7-10 days B. Meconium is the first stool and is usually yellow gold in color C. Vernix is a white, cheesy substance, predominantly located in the skin folds D. Pseudostrabismus found in newborns is treated by minor surgery.

C. Vernix is a white, cheesy substance, predominantly located in the skin fold

a 14-year old female client tells the nurse that she is concerned about the acne she has recently developed. Which recommendation should the nurse provide? A. Remove all blackheads and follow with an alcohol scrub B. Use medicated cosmetics only to help hide the blemishes. C. Wash the hair and skin frequently with soap and hot water. D. Encourage her to see a dermatologist as soon as possible.

C. Wash the hair and skin frequently with soap and hot water Washing the hair and skin with soap and hot water removes oil and debris from the skin and helps prevent and treat acne. Oily skin is especially bothersome during adolescence when hormones cause enlargement of sebaceous glands and increased glandular secretions which predispose the teenager to acne. (A) is contraindicated. Cosmetics ("medicated" or not) should be used sparingly to avoid further blocking of sebaceous gland ducts. (D) may be indicated at a later time, if healthcare recommendations are not successful.

A mother who is breastfeeding her baby receives instructions from the nurse. Which instruction is most effective to prevent nipple soreness? A. wear a cotton bra B. increase nursing time gradually C. correctly place the infant on the breast D. manually express a small amount of milk before nursing

C. correctly place the infant on the breast

A client with no prenatal care arrives at the labor unit screaming, "the baby is coming!" The nurse performs a vaginal examination that reveals the cervix is 3 cm dilated and 75% effaced. What additional information is most important for the nurse to obtain? A. gravidity and parity B. time and amount of last oral intake C. date of last normal menstrual period D. frequency and intensity of contractions

C. date of last normal menstrual period

A client at 32 weeks gestation is hospitalized with severe pregnancy-induced hypertension (PIH), and magnesium sulfate is prescribed to control symptoms. Which assessment finding would indicate that therapeutic drug level has been achieved? A. 4+ reflexes B. urinary output of 50 mL per hour C. a decrease in respiratory rate from 24 to 16 D. a decreased body temperature

C. a decrease in respiratory rate from 24 to 16

A newborn, whose mother is HIV positive, is scheduled for follow-up assessments. The nurse knows that the most likely presenting symptom for a pediatric client with AIDS is A. shortness of breath B. joint pain C. a persistent cold D. organomegaly

C. a persistent cold

The nurse is preparing a client with a term pregnancy who is in active labor for an amniotomy. What equipment should the nurse have available at the client's bedside? (select all that apply) A. litmus paper B. fetal scalp electrode C. a sterile glove D. an amnihook E. sterile vaginal speculum F. lubricant

C. a sterile glove D. an amnihook F. lubricant

The nurse is preparing a client with a term pregnancy who is in active labor for an amniotomy. What equipment should the nurse have available at the client's bedside? (select all that apply) A. Litmus paper B. fetal scalp electrode C. a sterile glove D. an amniotic hook E. sterile vaginal speculum F. a Doppler

C. a sterile glove D. an amniotic hook F. a Doppler

Surgery is being delayed for an infant with undescended testes. In collaboration with the healthcare provider and the family, which prescription should the nurse anticipate? A. a trial of adrenocorticotropic hormone injections B. Frequent stimulation of the cremasteric reflex C. a trial of human chorionic gonadotrophic hormone D. frequent warm baths to gently dilate the scrotal area.

C. a trial of human chorionic gonadotrophic hormone A trial of HCG (human chorionic gonadotrophic) hormone may aid in testicular descent, but does not replace surgical repair for true undescended testes. Undescended testes (cryptorchidism) may be found in the inguinal canal due to exaggerated cremasteric reflex. (A) is not indicated. Stimulation of the cremasteric reflex causes the testes to ascend rather than to descend in the scrotum. (D) may relax the cremasteric muscle, but may not cause the testes to descend.

A 16 year old is brought to the emergency department with a crushed leg after falling off a horse. the adolescent's last tetanus toxoid booster was received eight year ago. What action should the nurse take? A. dispense a tetanus antitoxin B. prepare human tetanus immune globulin C. administer tetanus toxoid booster D. delay the tetanus toxoid booster until due

C. administer tetanus toxoid booster After the completion of the initial tetanus immunization schedule, the recommended booster for an adolescent or adult is every 10 years or less is a traumatic injury occurs that is contaminated by dirt, feces, soil, or saliva, such as puncture or crushing injuries, avulsions, wounds from missles, burns, or frostbite. the adolescent's injury is considered a contaminated wound requiring prophylactic therapy, so the tetatnus toxoid booster should be administered.

A hospitalized 16-year old male refuses all visits from his classmates because he is concerned about his distorted appearance. To increase the client's social interaction, what intervention is best for the nurse to initiate? A. encourage the client to use a hand-held video game that is popular with all his friends B. assign a 25 year old female nursing student to offer support to the client C. arrange for an internet connection in the client's room for email communication D. encourage the client's mother to arrange a surprise get together in the cafeteria.

C. arrange for an internet connection in the client's room for email communication Body image and peer acceptance are key concerns for the adolescent. (C) allows for social interaction without face to face contact, thus protecting his self-image while also promoting social interaction. (A) does not promote social interaction. (B) does not encourage interaction with his own peer group, which is of greater import1ance. (D) does not respect the client's concern about his body image.

A newborn infant is brought to the nursery from the bathing suite. The nurse notices that the infant is breathing satisfactorily but appears dusky. What action should the nurse take first? A. notify the pediatrician B. suction the infant's nares, then the oral cavity C. check the infant's oxygen saturation rate D. position the infant on the right side.

C. check the infant's oxygen saturation rate

The nurse observes a 4-year old boy in a daycare setting. Which behavior should the nurse consider normal for this client? A. Has a temper tantrum when told he must share his toys. B. plays by himself most of the day C. demonstrates aggressiveness by boasting when telling a story D. Begins to cry and is fearful when separated from his parents.

C. demonstrates aggressiveness by boasting when telling a story Four-year old children are aggressive in their behavior and enjoy "tale telling". Behaviors in (A and D) are typical of toddlers. The play of a preschooler is cooperative, so playing alone is not typical.

A 15 year old girl tells the school nurse that all of her friends have started their periods and she feels abnormal because she has not. Which response is best for the nurse to provide? A. refer the adolescent to the healthcare provider for a pregnancy screen B. schedule a conference with her parents to recommend hormone therapy C. explain that menarche varies and occurs between the ages of 12 and 18 years. D. suggest that she use diversions to help her not worry about delayed menarche

C. explain that menarche varies and occurs between the ages of 12 and 18 years. The nurse should provide a factual and reassuring explanation that focuses on individual variations of menarche, which can normally occur between 12 and 18 years of age. (A) does not address the adolescent's concern and is judgmental. Menarche is influenced by hereditary general health and nutritional status, so (B) is not indicated. (D) dismisses the adolescent's concerns and does not offer factual information.

The nurse is teaching breastfeeding to prospective parents in a childbirth education class. Which instruction should the nurse include as content in the class? A. begin as soon as your baby is born to establish a four-hour feeding schedule B. resting helps with milk production. Ask that your baby be fed at night in the nursery C. feed your baby every 2 to 3 hours or on demand, whichever comes first. D. do not allow your baby to nurse any longer than the prescribed number of minutes

C. feed your baby every 2-3 hours or on demand, whichever comes first

The nurse is developing a plan of care for a 3-year old who is scheduled for a cardiac catheterization. To assist in decreasing anxiety for the child on the day of the procedure, which intervention is best for the nurse to implement? A. Reassure the parents that 3-year olds are cooperative and therefore are less likely to be anxious. B. Obtain a video film of a cardiac catheterization to show to the child prior to the procedure C. give the child a ride on a gurney to visit the cardiac catheterization lab and meet a nurse who works there. D. Obtain a cardiac catheter and demonstrate the procedure by pretending to put the catheter in a doll or stuffed animal.

C. give the child a ride on a gurney to visit the cardiac catheterization lab and meet a nurse who works there. Familiarizing the child and mother with the department will help decrease anxiety of the child and mother (who may have more anxiety than the child). Three is a difficult age to undergo a procedure that requires cooperation. Restraints and possibly sedation may be required. At three, the child is too young to understand why this must be done. (B) is not indicated and (D) is not indicated because it is likely to be interpreted as painful.

A 12-month old boy is admitted with a respiratory infection and possible pneumonia. He is placed in a mist tent with oxygen. Which nursing intervention has the greatest priority for this infant? A. give small, frequent feedings of fluids B. accurately chart observations regarding breath sounds C. have a bulb syringe readily available to remove secretions D. encourage older siblings to visit

C. have a bulb syringe readily available to remove secretions A patient airway has the highest priority. Humidification will liquefy the nasal secretions thereby increasing the amount of secretions and making (C) the highest priority. (A) maintains hydration and prevents, but an open airway has the highest priority. (B) is important for evaluations of therapy. When asked "priority" questions, remember Maslow. Physical needs usually have a higher priority than psychosocial needs. An open airway is the highest physiological need

At 14 weeks gestation, a client arrives at the emergency center complaining of a dull pain in the right lower quadrant of her abdomen. The nurse obtains a blood sample and initiates an IV. Thirty minutes after admission, the client reports feeling a sharp abdominal pain and a shoulder pain. Assessment findings include diaphoresis, a heart rate of 120 bpm, and a blood pressure of 86/48. Which action should the nurse implement next? A. check the hematocrit results B. administer pain medication C. increase the rate of IV fluids D. monitor client for contractions

C. increase the rate of IV fluids. The client is demonstrating symptoms of blood loss, probably the result of an ectopic pregnancy, which occurs at approximately 14 weeks gestation when embryonic growth expands the fallopian tube causing its rupture and can result in hemorrhage and hypovolemic shock. Increasing the IV infusion rate provides intravascular fluid to maintain blood pressure.

The nurse is assessing a 2-year old. What behavior indicates that the child's language development is within normal limits? A. Is able to name four colors B. can count five blocks C. is capable of making a three word sentence D. half of child's speech is understandable.

C. is capable of making a three word sentence A toddler 1-3 ye.ars old is capable of making two to three word sentences. Other options listed represent different age levels

The nurse assesses a client admitted to the labor and delivery unit and obtains the following data: dark red vaginal bleeding, uterus slightly tense between contractions, BP 110/68, FHR 110 bpm, cervix 1 cm dilated and uneffaced. Based on these assessment findings, what intervention should the nurse implement? A. insert an internal fetal monitor B. assess for cervical changes q1h C. monitor for bleeding from IV sites D. perform Leopold's maneuvers

C. monitor for bleeding from IV sites. This client is presenting with signs of placental abruption. Disseminated intravascular coagulation (DIC) is a complication of placental abruptio, characterized by abnormal bleeding.

A client at 30-weeks gestation, complaining of pressure over the pubic area, is admitted for observation. She is contracting irregularly and demonstrates underlying uterine irritability. Vaginal examination reveals that her cervix is closed, thick and high. Based on this data, which intervention should the nurse implement first? A. provide oral hydration B. have a complete blood count (CBC) drawn C. obtain a specimen for urine analysis D. place the client on strict bedrest

C. obtain a specimen for urine analysis. This should be done first because preterm clients with uterine irritability and contractions are often suffering from a UTI, and this should be ruled out first.

The nurse must prevent a 2-year old with severe eczema on the face, neck and scalp from scratching the affected areas. Which nursing intervention is most effective in preventing further excoriation due to the pruritis? A. obtain gloves for the child's hands B. apply finger cots on the child's fingers C. place elbow restraints on the child's arms D. apply soft restraints to the child's wrists

C. place elbow restraints on the child's arms Elbow restraints prevent arm flexion and scratching of involved areas, but do not inhibit use of the hands for play activities. (A and B) can be easily removed by the child and would restrict hand movement. (D) would be ineffective in preventing the child from scratching because the upper body could be moved within reach of restrained hands, and would also create the greatest restriction of hand movement.

Twenty minutes after a continuous epidural anesthetic is administered, a laboring client's blood pressure drops from 120/80 to 90/60. What action will the nurse take? A. notify the healthcare provider or anesthesiologist immediately. B. continue to assess the blood pressure q5 minutes C. place the woman in a lateral position D. turn off the continuous epidural

C. place the woman in a lateral position. The nurse should immediately turn the woman to a lateral position, place a pillow or wedge under the right hip to deflect the uterus, increase the rate of the main line IV infusion, and administer oxygen by face mask at 10-12 L/min. If the blood pressure remains low, especially if it further decreases ,the anesthesiologist/healthcare provider should be notified immediately. Turning off the continuous epidural may also be warranted, but such action is based on hospital protocol.

The nurse observes a new mother is rooming-in and caring for her newborn infant. Which observation indicates the need for further teaching? A. cuddles the baby close to her B. rocks and soothes the infant in her arms C. places the infant prone in the bassinet D. wraps the baby in a warm blanket after bathing

C. places the infant prone in the bassinet. This is associated with an increased incidence of sudden infant death syndrome (SIDS)

An infant is born with a ventricular septal defect (VSD) and surgery is planned to correct the defect. The nurse recognizes that surgical correction is designed to achieve which outcome? A. stop the flow of unoxygenated blood into systemic circulation B. increase the flow of unoxygenated blood to the lungs. C. prevent the return of oxygenated blood to the lungs. D. reduce peripheral tissue hypoxia and nailbed clubbing.

C. prevent the return of oxygenated blood to the lungs. Closure of VSDs stops oxygenated blood from being shunted from the left ventricle to the right ventricle. VSDs are acyanotic defects, which means that no unoxygenated blood enters the systemic circulation. (D) is common with Tetrology of Fallot, which is a cyanotic defect.

In developing a teaching plan for a 5 year old child with diabetes, which component of diabetic management should the nurse plan for the child to manage first? A. food planning and selection B. administering insulin injections C. process of glucose testing D. drawing up the correct insulin dose

C. process of glucose testing Developmentally a 5 year old has the cognitive and psychomotor skills to use a glucometer and to read the number (it is especially helpful if the nurse presents this activity as a game). (A, B, and D) require more advanced cognitive and psychomotor skills and have greater potential for errors.

A couple, concerned because the woman has not been able to conceive, is referred to a healthcare provider for a fertility workup and a hysterosalpingography is scheduled. Which complaint would indicate to the nurse that that woman's fallopian tubes are patient? A. back pain B. abdominal pain C. shoulder pain D. leg cramps

C. shoulder pain

During administration of a blood transfusion, a child complains of chills, headache and nausea. Which action should the nurse implement? A. start another IV of dextrose solution and stay with the child B. continue the tranfusion and monitor the child's vital signs C. stop the infusion immediately and notify the healthcare provider D. slow the transfusion and assess for cessation of symptoms

C. stop the infusion immediately and notify the healthcare provider The child is exhibiting signs of a reaction to the blood transfusion. The blood transfusion should be stopped immediately and the healthcare provider notified. After the transfusion is discontinued, IV access should be maintained with fluids that do not introduce and more cellular products. (B and D place the child at risk for further blood reactions

A 26-year old, gravida 2, para 1 client is admitted to the hospital at 28 weeks gestation in preterm labor. She is given 3 doses of terbutaline sulfate (Brethine) 0.25mg SQ to stop her labor contractions. The nurse plans to monitor for which primary side effect of terbutaline sulfate? A. drowsiness and bradycardia B. depressed reflexes and increased respirations C. tachycardia and a feeling of nervousness D. a flushed, warm feeling and a dry mouth

C. tachycardia and a feeling of nervousness

A 23 year old client who is receiving Medicaid is pregnant with her first child. Based on knowledge of the statistics related to infant mortality, which plan should the nurse implement with this client? A. refer the client to a social worker to arrange for home care B. recommend prenatal care from an obstetrician, not a nurse midwife C. teach the client why keeping prenatal care appointments is important D. Advise the client that neonatal intensive care may be needed

C. teach the client why keeping prenatal care appointments is important

The nurse should encourage the laboring client to begin pushing when A. there is only an anterior or posterior lip of cervix left. B. the client describes the need to have a bowel movement. C. the cervix is completely dilated D. the cervix is completely effaced

C. the cervix is completely dilated

The nurse is assessing the umbilical cord of a newborn. Which finding constitutes a normal finding? A. two vessels: one artery and one vein B. two vessels: two arteries and no veins C. three vessels: two arteries and one vein D. three vessels: Two veins and one artery

C. three vessels: two arteries and one vein

A child falls on the playground and is brought to the school nurse with a small laceration on the forearm. Which action should the nurse implement first? A. slowly pour hydrogen peroxide over the open wound B. apply ice to the area before rinsing with cold water C. wash the wound gently with mild soap and water D. gently cleanse with a sterile pad using povidone-iodine

C. wash the wound gently with mild soap and water A small superficial laceration to the skin should be washed gently with mild soap and water for several minutes, followed by thorough rinsing. (A and D) are antiseptics that can be traumatic (painful) when cleaning fresh, open wounds. Applying ice may reduce or prevent further edema, but the wound should be washed with mild soap and water first.

A client at 32 weeks gestation comes to the prenatal clinic with complaints of pedal edema, dyspnea, fatigue and a moist cough. Which question is most important for the nurse to ask this client? A. "Which symptom did you experience first?" B. "Are you eating large amounts of salty foods?" C. "Have you visited a foreign country recently?" D. "Do you have a history of rheumatic fever?"

D. "Do you have a history of rheumatic fever?" Clients with a history of rheumatic fever may develop mitral valve prolapse, which increases the risk for cardiac decompensation due to the increased blood volume that occurs during pregnancy, so obtaining information about this client's health history is a priority.

A client who is in the second trimester tells the nurse that she wants to use herbal therapy. Which response is best for the nurse to provide? A. "Herbs are a cornerstone of good health to include in your treatment." B. "Touch is also therapeutic in relieving discomfort and anxiety." C. "Your healthcare provider should direct treatment options for herbal therapy." D. "It is important that you want to take part in your care."

D. "It is important that you want to take part in your care."

A 30-year old multiparous woman who has a 3 year old boy and a newborn girl tells the nurse, "My son is so jealous of my daughter. I don't know who I'll ever manage both children when I get home." How should the nurse respond? A. "Tell the older child that he is a big boy now and should love his new sister." B. "Ask friends and relatives not to bring gifts to the older sibling because you do not want to spoil him." C. "Let the older child stay with his grandparents for the first 6 weeks to allow him to adjust to the newborn." D. "Regression in behaviors in the older child is a typical reaction so he needs attention at this time."

D. "Regression in behaviors in the older child is a typical reaction so he needs attention at this time." Preschool-aged children frequently regress in habits or behaviors, such as toileting and sleep habits, as a method of seeking attention so the parents should distribute their attention between the children and include the preschooler during infant care.

A new mother is afraid to touch her baby's head for fear of hurting the "large soft spot". Which explanation should the nurse give to this anxious client? A. "Some care is required when touching the large soft area on top of your baby's head until the bones fuse together." B. "That's just an 'old wives' tale' so don't worry, you can't harm your baby's head by touching the soft spot. C. "The soft spot will disappear within 6 weeks and is very unlikely to cause any problems for your baby." D. "There's a strong, tough membrane there to protect the baby so you need not be afraid to wash or comb his/her hair."

D. "There's a strong, tough membrane there to protect the baby so you need not be afraid to wash or comb his/her hair." The anterior fontanel or "large soft spot" normally closes at 12-18 months of age.

A pregnant woman comes to the prenatal clinic for an initial visit. In reviewing her childbearing history, the client indicates that she has delivered premature twins, one full-term baby, and has had no abortions. Which GTPAL should the nurse document in the client's record? A. 31203 B. 41203 C. 21212 D. 31103

D. 31103. The client has been pregnant 3 times including the current pregnancy (G3); She had one full-term infant (T1); She also had a preterm (P1) twin pregnancy (a multifetal gestation is considered one birth when calculating parity); There were no abortions (A0), so this client has a total of 3 living children.

The nurse is preparing to give an enema to a laboring client. Which client would require the most caution when carrying out this procedure? A. a gravida 6, para 5 who is 38 years of age an in early labor B. a 37 week primigravida who presents at 100% effacement, 3 cm cervical dilation and a -1 station. C. A gravida 2, para 1 who is at 1 cm cervical dilation and a 0 station admitted for induction of labor due to post dates D. A 40-wk primigravida who is at 6 cm dilation and the presenting part is not engaged

D. A 40-wk primigravida who is at 6 cm dilation and the presenting part is not engaged. When the presenting part is ballottable, it is floating out of the pelvis. In such a situation, the cord can descent before the fetus causing a prolapsed cord, which is an emergency situation.

The nurse is giving preoperative instructions to a 14-year old female client who is scheduled for surgery to correct a spinal curvature. Which statement by the client best demonstrates that learning has taken place? A. I will read all the literature you gave me before surgery. B. I have had surgery before when I broke my wrist in a bike accident, so I know what to expect. C. All the things people have told me will help me take care of my back. D. I understand that I will be in a body cast and I will show you how you taught me to turn.

D. I understand that I will be in a body cast and I will show you how you taught me to turn. Outcome of learning is best demonstrated when the client not only verbalizes an understanding, but also can provide a return demonstration. A 14-year old may or may not follow through with reading material and there is no way of measuring that way of learning. Have a previous surgery may help the client understand the surgical process, but wrist surgery is very different from spinal surgery and emergency surgery is different than elective surgery. In (C), the client may be saying what the nurse wants to hear, without expressing any real understanding of what to do after surgery.

An expectant father tells the nurse he fears that his wife "is losing her mind." He states she is constantly rubbing her abdomen and talking to the baby, and that she actually reprimands the baby when it moves too much. What recommendation should the nurse make to this expectant father? A. Reassure him that these are normal reactions to pregnancy and suggest that he discuss his concerns with the childbirth education nurse. B. Help him to understand that his wife is experiencing normal symptoms of ambivalence about the pregnancy and no action is needed. C. Ask him to observe his wife's behavior carefully for the next few weeks and report any similar behavior to the nurse at the next prenatal visit. D. Let him know that these behaviors are part of normal maternal/fetal bonding which occur once the mother feels fetal movement.

D. Let him know that these behaviors are part of normal maternal/fetal bonding which occur once the mother feels fetal movement.

A client who gave birth to a healthy 8 pound infant 3 hours ago is admitted to the postpartum unit. Which nursing plan is best in assisting this mother to bond with her newborn infant? A. encourage the mother to provide total care for her infant B. provide privacy so the mother can develop a relationship with the infant. C. encourage the father to provide most of the infant's care during hospitalization. D. Meet the mother's physical needs and demonstrate warmth toward the infant.

D. Meet the mother's physical needs and demonstrate warmth toward the infant.

Just after delivery, a new mother tells the nurse, "I was unsuccessful breastfeeding my first child, but I would like to try with this baby." Which intervention is best for the nurse to implement first? A. Assess the husband's feelings about his wife's decision to breastfeed their baby B. Ask the client to describe why she was unsuccessful with breastfeeding her last child C. Encourage the client to develop a positive attitude about breastfeeding to help ensure success D. Provide assistance to the mother to begin breastfeeding as soon as possible after delivery.

D. Provide assistance to the mother to begin breastfeeding as soon as possible after delivery. Infants respond to breastfeeding best when feeding is initiated in the active phase soon after delivery

Preoperative nursing care for a child with Wilm's tumor should include which intervention? A. gently percuss the abdomen for evidence of trapped air B. observe the abdomen for any noticeable discolorations C. apply cold compresses to the abdomen to reduce edema D. Put a sign on the bed reading, "DO NOT PALPATE ABDOMEN."

D. Put a sign on the bed reading, "DO NOT PALPATE ABDOMEN." Prevention of abdominal palpation minimizes the risk of rupturing the encapsulated tumor and subsequent metastasis. (A) is unnecessary and this action could traumatize the tumor in the same manner as palpation. (B and C) are incorrect since the abdomen is not discolored and cold compresses are not indicated.

After each feeding, a 3-day old newborn is spitting up large amounts of Enfamil Newborn Formula, a nonfat cow's milk formula. The pediatric healthcare provider changes the neonate's formula to Similac Soy Isomil Formula, a soy protein isolate based on infant formula. What information should the nurse provide to the mother about the newly prescribed formula? A. The new formula is a coconut milk formula used with babies with impaired fat absorption B. Enfamil Formula is a demineralized whey formula that is needed with diarrhea C. the new formula is a casein protein source that is low in pheynylalanine. D. Similac Soy Isomil Formula is a soy-based formula that contains sucrose.

D. Similac Soy Isomil Formula is a soy-based formula that contains sucrose. The nurse should explain that the newborn's feeding intolerance may be related to the lactose found in cow's milk formula and is being replaced with the soy-based formula that contains sucrose which is well-tolerated in infants with milk allergies and lactose intolerances

In developing a teaching plan for expectant parents, the nurse plans to including information about when the parents can expect the infant's fontanels to close. The nurse bases the explanation on knowledge that for the normal newborn, the A. anterior fontanel closes at 2-4 months and the posterior by the end of the first week B. anterior fontanel closes at 5-7 months and the posterior by the end of the second week C. anterior fontanel closes at 8-11 months and the posterior by the end of the first month D. anterior fontanel closes at 12-18 months and the posterior by the end of the second month.

D. anterior fontanel closes at 12-18 months and the posterior by the end of the second month.

A couple has been trying to conceive for 9 months without success. Which information obtained from the clients is most likely to have an impact on the couple's ability to conceive a child? A. Exercise regimen of both partners includes running 4 miles each morning B. history of having sexual intercourse 2-3x/wk. C. The woman's menstrual period occurs every 35 days D. They use lubricants with each sexual encounter to decrease friction

D. They use lubricants with each sexual encounter to decrease friction. The use of lubricants has the potential to affect fertility because some lubricants interfere with sperm motility.

A 2 year old child recently diagnosed with hemophilia A is discharged home. What information should the nurse include in a teaching plan about home care? A. Minimize interactive play with other children to lessen chances for injury B. give low-dose children's chewable aspirin in orange flavor for joint discomfort C. use a firm and dry toothbrush to clean teeth at least twice per day D. apply pressure and ice for bleeding while elevating and resting the extremity.

D. apply pressure and ice for bleeding while elevating and resting the extremity. Hemophilia, a blood disorder, causes joint bleeding which is treated with rest, ice, compression, and elevation (RICE)

The nurse is assessing a client who is having a non-stress test (NST) at 41 weeks gestation. The nurse determines that the client is not having contractions, the fetal heart rate (FHR) baseline is 144 bpm, and no FHR accelerations are occuring. What action should the nurse take? A. check the client for urinary bladder distension B. notify the healthcare provider of the nonreactive results C. have the mother stimulate the fetus to move D. ask the client if she has felt any fetal movement

D. ask the client if she has felt any fetal movement. An NST is used to determine fetal well-being and is often implemented when postmaturity is suspected. A 'reactive' NST occurs if the FHR accelerates 15 bpm for 15 seconds in response to the fetus' own movement, and is "nonreacctive" if no FHR acceleration occurs in response to fetal movement. The client should empty her bladder before starting the test, but bladder distention does not impede fetal movement. The client should be quizzed about fetal movement before determining that the NST is nonreactive. If no movement has occurred in the last 20-30 minutes, it is likely that the fetus is sleeping. Providing the mother with orange juice often wakes the infant, and then the NST should be conducted again.

A child is rescued from a burning house and brought to the emergency room with partial-thickness burns on the face and chest. Which action should the nurse implement first? A. insert an indwelling urinary catheter B. administer IV pain medication C. collect blood specimen for laboratory studies D. assess the child's respiratory status.

D. assess the child's respiratory status Assessing the airway and the respiratory status is the highest priority since burns to the face and chest place the child at risk for smoke inhalation injury and compromised airway

When assessing a client who is at 12-weeks gestation, the nurse recommends that she and her husband consider attending childbirth preparation classes. When is the best time for the couple to attend these classes? A. at 16 weeks gestation B. at 20 weeks gestation C. at 24 weeks gestation D. at 30 weeks gestation

D. at 30 weeks gestation

One hour after giving birth to an 8 pound infant, a client's lochia rubra has increased from small to large and her fundus is boggy despite massage. The client's pulse is 84 bpm and blood pressure is 156/96. The healthcare provider prescribes Methergine 0.2 mg IM x 1. What action should the nurse take immediately? A. give the medication as prescribed and monitor for efficacy B. encourage the client to breast feed rather than bottle feed C. have the client empty her bladder and then massage the fundus D. call the healthcare provider to question the prescription

D. call the healthcare provider to question the prescription. Methergine is contraindicated for clients with elevated blood pressure, so the nurse should contact the healthcare provider and question the prescription.

Which assessment finding should the nursery nurse report to the pediatric healthcare provider? A. blood glucose level of 45 mg/dl B. blood pressure of 82/45 C. non-bulging anterior fontanel D. central cyanosis when crying

D. central cyanosis when crying. An infant who demonstrates central cyanosis when crying is manifesting poor adaption to extrauterine life which should be reported to the healthcare provider for determination of a possible underlying cardiovascular problem.

When taking the health history of a child, the nurse knows that which finding is an early indication of hypothyroidism in children? A. hyperactive behavioral traits B. delay in the eruption of permanent teeth C. slow sexual development but within normal range D. cessation of growth in a child that had been normal.

D. cessation of growth in a child that had been normal. Since the thyroid gland is responsible for metabolism, cessation of growth which was previously within normal range, is the most common sign for hypothyroidism in children. The child with hypothyroidism is likely to be HYPOactive, not hyperactive. Although (B and C) may occur with hypothyroidism, they are late signs (not early indications) and are signs more often associated with a lack of growth hormone.

What action should the nurse implement to decrease the client's risk for hemorrhage after a cesarean section? A. monitor urinary output via an indwelling catheter B. assess the abdominal dressings for drainage C. give the Ringer's lactated infusion at 125 ml/hr. D. check the firmness of the uterus every 15 minutes

D. check the firmness of the uterus every 15 minutes. A client's risk for postpartal hemorrhage is decreased when the uterus is firm after delivery of the infant. Assessment of fundus consistency q15 min provides frequent intervals to stimulate the fundus to contract and prevent bleeding.

The nurse is assessing an 8-month old child who has a medical diagnosis of Tetrology of Fallot. Which symptom is this client most likely to exhibit? A. bradycardia B. machinery murmur C. weak pedal pulses D. clubbed fingers

D. clubbed fingers Tetrology of Fallot, a cyanotic heart defect, causes clubbing of fingers and toes due to tissue hypoxia. Tachycardia not bradycardia is a manifestation of congenital heart disease. Machinery murmur is a classic sign of ventricular septal defect. Weak pedal pulses are characteristic of coarctation of the aorta

A 2-year old with gastro-esophageal reflux has developed a fear of eating. What instruction should the nurse include in the parents' teaching plan? A. invite other children home to share meals B. accept that he will eat when he is hungry C. reward the child with a nap after eating D. consistently follow a set mealtime routine.

D. consistently follow a set mealtime routine. A 2-year old child is comforted by consistency. (A) is contraindicate because 2 year olds may participate in parallel activities with other children but are too young to feel comfort and support by the presence of other children when anxious or afraid. (B) may or may not be true and does not address the child's fears. The child with reflux should remain upright at least 2 hours after eating to reduce symptoms.

The mother of a 4 year old child asks the nurse what she can do to help her other children cope with their sibling's repeated hospitalizations. Which is the best response that the nurse should offer? A. inform the parent that the child is too young to visit the hospital B. suggest that the child visit a grandmother until the sibling returns home. C. ask the mother if the child asks when the sibling will be discharged D. encourage the mother to have the children visit the hospitalized sibling.

D. encourage the mother to have the children visit the hospitalized sibling. Needs of a sibling will be better met with factual information and contact with the ill child, so sibling visitation should be encouraged. Parents are experts on their children and should determine when their children are old enough to visit in the hospital. Separation from family and home may intensify fear and anxiety. Children may have difficulty expressing questions so the support of parents and other caregivers are needed to help alleviate their fears.

What preoperative nursing intervention should be included in the plan of care for an infant with pyloric stenosis? A. Monitor for signs of metabolic acidosis. B. estimate the quantity of diarrhea stools. C. place in a supine position after feeding D. observe for projectile vomiting.

D. observe for projectile vomiting. Projectile vomiting which contributes to metabolic alkalosis, is the classic sign of pyloric stenosis. Estimating the quantity of diarrhea stools is not indicated. Placing the child in a supine position is dangerous due to the potential for aspiration with frequent vomiting.

An off-duty nurse finds a woman in a supermarket parking lot delivery an infant while her husband is screaming for someone to help his wife. Which intervention has the highest priority? A. use a thread to tie off the umbilical cord. B. provide as much privacy as possible for the woman. C. reassure the husband and try to keep him calm D. put the newborn to breast.

D. put the newborn to breast. Putting the newborn to breast will help contract the uterus and prevent a postpartum hemorrhage - this intervention has the highest priority.

An 18-month old is admitted to the hospital with possible Hirschsprung's disease. When obtaining a nursing history, the nurse asks about bowel habits. What describes the disease? A. foul-smelling and fatty B. bile-colored and watery C. semi-solid and yellow D. ribbon-like and brown

D. ribbon-like and brown Hirschsprung's disease is a mechanical obstruction caused by inadequate motility in a part of the intestines. The condition results from failure of ganglion cells to migrate craniocaudally along the GI tract during gestation. the lack of peristalsis in the affected bowel segment causes constipation and small diameter, brown-colored stools. (A) is associated with cystic fibrosis. (B) is common in gastroenteritis. (C) is normal for breastfed neonates.

A child with cystic fibrosis is having stools that float and are foul smelling. Which descriptive term should the nurse use to document the finding? A. diarrhea B. rhinorrhea C. galactorrhea D. steatorrhea

D. steatorrhea Steatorrhea is defined as stools with an abnormally high fat content that are usually foul smelling and float on water

When planning the care for a child who has had a cleft lip repair, the nurse knows that crying should be minimized because it A. increases salivation B. increases the respiratory rate C. leads to vomiting D. stresses the suture line

D. stresses the suture line Prevention of stress on the lip suture line is essential for optimum healing and the cosmetic appearance of a cleft lip repair. Although crying also causes increased salivation, increased respiratory rate and may lead to vomiting, these conditions do not create a problem for the child with a cleft lip repair.

While breastfeeding, a new mother strokes the top of her baby's head and asks the nurse about the baby's swollen scalp. The nurse responds that the swelling is caput succadeaneum. Which additional information should the nurse provide this new mother? A. the infant should be positioned to reduce the swelling B. the swelling is a subperiosteal collection of blood C. the pediatrician will aspirate the blood if it gets larger D. the scalp edema will subside in a few days after birth.

D. the scalp edema will subside in a few days after birth. Caput succadeaneum is edema of the fetal scalp that crosses over the suture lines and is caused by pressure on the fetal head against the cervix during labor. It will subside in a few days after birth without treatment.

The mother of a 2-year old boy consults the nurse about her son's increased tempter tantrums. The mother states, "Yesterday he threw a fit in the grocery store, and I did not know what to do. I was embarrassed. What can I do if this occurs again?" Which recommendation is best for the nurse to provide this mother? A. paddle him gently as soon as the behavior is initiated B. immediately put him in "time-out" C. quietly remind him that others are watching him D. walk away from him and ignore the behavior

D. walk away from him and ignore the behavior The best approach for a toddler is to ignore the attention-seeking behavior. The parent should be somewhat nearby within view of the child but should avoid reinforcing the behavior in any way. Tantrums can sometimes be avoided by talking to the child before the situation occurs. (A, B, and C) would all provide attention for the inappropriate behavior.

A client who delivered an infant an hour ago tells the nurse that she feels wet underneath her buttock. The nurse notes that both perineal pads are completely saturated and the client is lying in a 6 inch diameter pool of blood. Which action should the nurse implement next? A. cleanse the perineum B. obtain a blood pressure C. palpate the firmness of the fundus D. inspect the perineum for lacerations.

Palpate the firmness of the fundus. A firm fundus is needed to control bleeding from the placental site of attachment on the uterine wall. The nurse should first assess for firmness and massage the fundus as indicated.

A woman who had a miscarriage 6 months ago became pregnant. Which instruction is most important for the nurse to provide this client? A. Elevate lower legs while resting B. increase caloric intake by 200-300 calories per day C. increase water intake to 8 full glasses per day D. take prescribed multivitamin and mineral supplements

take prescribed multivitamin and mineral supplements


Related study sets

CA RDA Test Bank - Specialty Dentistry

View Set

Seasonal Influenza Vaccination Training (2hr)

View Set

Chapter 1: Insurance Terms and Related Concepts

View Set

Life and Health Insurance Quiz Questions

View Set

Free Enterprise Economy: Definition & Examples

View Set